Musculoskeletal System Pathology 2
Time limit: 0
Quiz Summary
0 of 38 Questions completed
Questions:
Information
You have already completed the quiz before. Hence you can not start it again.
Quiz is loading…
You must sign in or sign up to start the quiz.
You must first complete the following:
Results
Quiz complete. Results are being recorded.
Results
0 of 38 Questions answered correctly
Your time:
Time has elapsed
You have reached 0 of 0 point(s), (0)
Earned Point(s): 0 of 0, (0)
0 Essay(s) Pending (Possible Point(s): 0)
Average score |
|
Your score |
|
Categories
- Not categorized 0%
- NBME 7 BLOCK 4 0%
Would you like to submit your quiz result to the leaderboard?
Loading
Pos. | Name | Entered on | Points | Result |
---|---|---|---|---|
Table is loading | ||||
No data available | ||||
- 1
- 2
- 3
- 4
- 5
- 6
- 7
- 8
- 9
- 10
- 11
- 12
- 13
- 14
- 15
- 16
- 17
- 18
- 19
- 20
- 21
- 22
- 23
- 24
- 25
- 26
- 27
- 28
- 29
- 30
- 31
- 32
- 33
- 34
- 35
- 36
- 37
- 38
- Current
- Review
- Answered
- Correct
- Incorrect
-
Question 1 of 38
1. Question
A 22-year-old man comes to the office due to 3 months of progressive back pain. He has also had intermittent subjective fever. The patient emigrated from Nepal for a master’s program 6 months ago. He has no other medical conditions, takes no medication, and does not use tobacco, alcohol, or illicit drugs. Temperature is 38.3 C (100.9 F), blood pressure is 122/78 mm Hg, and pulse is 84/min. BMI is 19 kg/m2. Tenderness is noted over the lumbar spine; flexion, extension, and rotation of the spine are limited due to pain and muscle spasm. MRI of the lower spine reveals partial destruction of the anterior portion of the L1-L3 vertebral bodies and a fluid collection beneath the anterior longitudinal ligament. Which of the following is the most likely diagnosis?
CorrectIncorrect -
Question 2 of 38
2. Question
A 70-year-old woman is brought to the emergency department due to acute right hip pain after a fall. She was walking in her house when she tripped on a floor rug and fell, landing on her hip. The patient has been unable to bear weight on the right leg since the fall and had to call for emergency transport to the hospital. She has no significant medical history and does not use tobacco, alcohol, or illicit drugs. The patient’s only medication is an over-the-counter multivitamin. Family history is notable for a hip fracture in her mother. Blood pressure is 150/90 mm Hg and pulse is 112/min. The patient is in moderate pain and her right leg appears shortened and externally rotated. There is tenderness and swelling of the right hip. Radiographs reveal a right femoral neck fracture. Which of the following changes in bone structure is most likely responsible for this patient’s condition?
CorrectIncorrect -
Question 3 of 38
3. Question
A 57-year-old man comes to the emergency department due to a 1-day history of pain and swelling in his right knee. The pain is severe, and he has had difficulty bearing weight on the right leg. He previously felt well. Past medical history is notable for hypertension and seasonal allergies. The patient does not use tobacco and drinks 1 or 2 alcoholic beverages a week. On examination, the right knee is red and swollen. He has pain with passive range of motion at the knee. The knee ligaments show no significant laxity. Synovial fluid obtained from the involved joint shows a white blood cell count of 25,000/mm3, with a neutrophilic predominance, and numerous crystals (shown below).
These crystals are most likely composed of which of the following substances?
CorrectIncorrect -
Question 4 of 38
4. Question
A 29-year-old woman is evaluated for severe left hip pain after twisting her leg. She has bony deformities of the lower extremities, and her mobility is limited at baseline. The patient has a history of hyperthyroidism, which was managed with radioiodine therapy. Menarche was at age 7, and she has regular 30-day cycles. Physical examination shows large, hyperpigmented macules with irregular borders located on the left shoulder, left side of the neck, and left buttock. X-ray findings are shown in the image below.
Which of the following is the most likely diagnosis?
CorrectIncorrect -
Question 5 of 38
5. Question
A 34-year-old man comes to the office due to progressive low back pain for several years. The patient reports that the stiffness and pain are worse in the morning but gradually improve during the day. Lately, he has been taking over-the-counter ibuprofen, which provides some relief. Laboratory evaluation shows a positive HLA-B27 antigen. X-ray imaging reveals fusion of the sacroiliac joints. Periodic measurement of which of the following is most helpful to monitor disease progression in this patient?
CorrectIncorrect -
Question 6 of 38
6. Question
A 70-year-old man comes to the clinic due to intermittent leg pain and difficulty walking. He describes the pain as mild-to-moderate, deep, and lasting throughout the day. The patient has a history of osteoarthritis in his hands for which he takes over-the-counter nonsteroidal anti-inflammatory drugs, but says that he never has had problems with his knees. Physical examination shows point tenderness over the right tibia. Laboratory studies show elevated serum alkaline phosphatase. Bone biopsy shows haphazardly oriented segments of lamellar bone with prominent cement lines. The initial phase of this patient’s disorder involves increased activity of which of the following cell types?
CorrectIncorrect -
Question 7 of 38
7. Question
A 47-year-old woman comes to the physician with progressive joint pain and swelling in her hands for the past several months. She also has easy fatigability that has gradually worsened over the same period. Morning activities are especially difficult due to stiffness lasting 1 to 2 hours after waking. Examination shows warmth, swelling, and tenderness involving the proximal interphalangeal joints, metacarpophalangeal joints, and wrists bilaterally. A blood sample is obtained for laboratory analysis. Autoantibodies against which of the following components are most specific for this patient’s condition?
CorrectIncorrect -
Question 8 of 38
8. Question
A 45-year-old woman comes to the clinic with a 3-month history of persistent shoulder and back pain and easy fatigability. She also has pain and stiffness in all her muscles that worsens in the morning and evening. She does not participate in any regular exercise because her pain is exacerbated with activity. She works as a computer programmer and reports some difficulty concentrating on her job. Past medical history is significant for depression and gastroesophageal reflux disease, but she is not currently taking any medications. Physical examination shows normal range of motion and 5/5 muscle strength in both upper and lower extremities. Soft tissue tenderness is present at several locations bilaterally above and below the waist. Which of the following is the most likely diagnosis?
CorrectIncorrect -
Question 9 of 38
9. Question
A 62-year-old woman comes to the office due to a 3-month history of painful, swollen wrists and knees. She also has joint stiffness, which is worse upon wakening and limits her daily activities. Review of systems is positive for fatigue. The patient’s only other medical condition is hypothyroidism for which she takes levothyroxine. She smoked a pack of cigarettes daily for 20 years and quit 15 years ago. Vitals signs are within normal limits. Physical examination shows symmetric, moderate swelling of the wrists and knees. The joints are tender and warm. Range of motion is intact but painful. Plain radiographs of the symptomatic joints show joint space narrowing and marginal erosions. Which of the following cytokines are primarily involved in the pathogenesis of this patient’s joint destruction?
CorrectIncorrect -
Question 10 of 38
10. Question
A 34-year-old woman comes to the office with several months of vision problems, difficulty chewing, and trouble speaking. The symptoms fluctuate, but the patient has noticed that they are worse after a long day. She works as a telemarketer and has been putting in increasingly long hours to meet her quotas. Regarding her symptoms, she says, “It’s probably just exhaustion; an ice pack to my face seems to help.” During physical examination, the patient is asked to keep her eyes focused on a spot on the ceiling, and after 2 minutes, her vision becomes blurry. The remainder of the examination is normal. Further evaluation shows that the patient’s symptoms are reversible on administration of an acetylcholinesterase inhibitor. The pathogenesis of this patient’s disease is most similar to which of the following conditions?
CorrectIncorrect -
Question 11 of 38
11. Question
A 64-year-old man comes to the emergency department due to acute knee pain. He developed redness, swelling, and severe pain affecting the right knee the day before his arrival. The patient has had 3 similar episodes in the past year that resolved quickly with over-the-counter analgesics. He has a history of polycythemia vera but has been noncompliant with phlebotomy treatments. Vital signs and cardiopulmonary examinations are normal. The right knee is red and warm. There is severe pain with movement at the joint; the other joints are grossly normal. Needle aspiration of the joint is performed. Which of the following is the most likely finding on analysis of this patient’s synovial fluid?
CorrectIncorrect -
Question 12 of 38
12. Question
A 43-year-old man comes to the office with muscle weakness. His symptoms began 6 weeks ago and are worst in the hips and shoulders. The weakness has had a progressive course so that he is now having difficulty rising from chairs and combing his hair. The patient has had to reduce his regular exercise regimen due to weakness and has requested a modified work schedule for his job as a building maintenance manager. He has no history of spinal injury and no neck pain. Past medical history is insignificant, and the patient takes no medications. Muscle biopsy reveals major histocompatibility complex class I molecule overexpression on the sarcolemma with CD8+ lymphocyte infiltration. Which of the following is the most likely cause of this patient’s condition?
CorrectIncorrect -
Question 13 of 38
13. Question
A 35-year-old man comes to the clinic with acute right knee pain and swelling. His symptoms have been present for a week and have moderately worsened over this period. The patient is able to bear weight but has significant pain when climbing stairs or walking for extended distances. He attempted treatment with ibuprofen, which provided prompt but only temporary relief. Past medical history is unremarkable, though the patient was seen by his primary care provider for a diarrheal illness 2 weeks before onset of the current symptoms. Examination shows a moderate-sized effusion at the right knee. Cultures of a joint aspirate reveal no bacteria. Which of the following is most likely associated with this patient’s joint symptoms?
CorrectIncorrect -
Question 14 of 38
14. Question
A 54-year-old man is admitted to the hospital following a high-speed motor vehicle accident. He is found to have right leg long bone fractures requiring open reduction/internal fixation. After the patient is stabilized, he is transferred to a skilled nursing facility for an extended rehabilitation period. During his stay at the facility, his right leg is immobilized for several weeks. The patient’s management is complicated by decreased nutritional intake due to side effects from opioid pain medications. Measurement of his thigh following removal of casting materials finds a 2-centimeter decrease in diameter during the time of immobilization. This change in body measurement is due to a pathologic process similar to that seen in which of the following?
CorrectIncorrect -
Question 15 of 38
15. Question
A 24-year-old woman comes to the office at 12 weeks gestation for prenatal counseling. She is 120 cm (3 ft 11 in) tall with short upper and lower extremities but normal torso length. Physical examination is also significant for depression of the nasal bridge and a bulging forehead. Her husband is phenotypically normal and has no medical problems. The patient knows that there is a 50% chance of passing on her condition to the fetus. An abnormality involving which of the following cells is most likely responsible for this patient’s features?
CorrectIncorrect -
Question 16 of 38
16. Question
A 48-year-old previously healthy woman comes to the office due to progressively worsening muscle weakness for the past 2 months. The patient has difficulty with activities such as climbing stairs, getting up from chairs, and placing dishes in overhead cabinets. She also reports a 4.5-kg (10-lb) unintentional weight loss and occasional abdominal discomfort over the same interval. Physical examination shows weakness of the shoulder and hip girdle muscles. Other examination findings are shown in the image below.
Further evaluation of this patient is most likely to reveal which of
the following underlying conditions?
CorrectIncorrect -
Question 17 of 38
17. Question
A 55-year-old man comes to the office for evaluation of chronic muscle weakness. Over the past several months, he has had increasing difficulty walking up stairs and lately has had trouble removing objects out of the overhead cabinets in his kitchen. Temperature is 36.7 C (98 F), blood pressure is 125/80 mm Hg, and pulse is 78/min. On examination, the patient has symmetric proximal muscle weakness and mild muscle tenderness. There is no rash. Muscle biopsy reveals an endomysial mononuclear infiltrate and patchy muscle fiber necrosis. An autoantibody directed against which of the following antigens is most likely to be seen in this patient?
CorrectIncorrect -
Question 18 of 38
18. Question
A 35-year-old woman comes to the clinic due to severe heartburn that is resistant to over-the-counter antacids. The patient has no known medical problems and takes no other medications. She occasionally has a glass of wine with dinner but does not use tobacco or illicit drugs. Physical examination shows scattered telangiectasias on the face, several ulcers at the tips of the fingers, and small calcium deposits in the soft tissues of the hands and elbows. Which of the following processes is the most likely cause of this patient’s heartburn?
CorrectIncorrect -
Question 19 of 38
19. Question
A 46-year-old woman comes to the office due to dysphagia. The patient has had difficulty swallowing both liquids and solid foods as well as frequent episodes of regurgitation and cough while eating. She has also had weakness in her extremities and a rash. The patient has no prior medical problems and takes no medications. Vital signs are within normal limits. Physical examination shows an erythematous eruption on the upper eyelids. The oropharynx is clear and cardiopulmonary and abdominal examinations are unremarkable. She has difficulty lifting her arms above her head and standing from a sitting position. Which of the following is the most likely cause of this patient’s dysphagia?
CorrectIncorrect -
Question 20 of 38
20. Question
A 68-year-old man comes to the office due to an enlarging mole on his right forearm. The patient is a retired farmer and received a significant amount of sun exposure over the course of his life. On examination, he has a black-brown macular lesion on the dorsum of his right forearm measuring approximately 1 cm in diameter with an irregular border. Excisional biopsy is performed and histopathology reveals malignant melanoma. Immunohistochemical analysis indicates that the malignant cells have decreased integrin expression. These cells are most likely to exhibit poor adhesion to which of the following components of the extracellular matrix?
CorrectIncorrect -
Question 21 of 38
21. Question
An 18-year-old man is evaluated due to 3 months of lower extremity pain. The pain is persistent and present both day and night. He has no fever, chills, or history of leg trauma. Imaging shows an expansile lesion of the distal femur. Surgical resection is performed, and the specimen is shown below.
Mutations involving which of the following genes are most likely present in this patient?
CorrectIncorrect -
Question 22 of 38
22. Question
A 15-year-old boy is brought to the emergency department due to hemoptysis. He has a history of amputation of the right lower extremity due to bone cancer. Chest imaging reveals a lung mass. Excisional biopsy of the mass shows sheets of uniform, round, small cells slightly larger than lymphocytes with scant, clear cytoplasm. The cellular deposits are interrupted by vascular fibrous septa, with areas of hemorrhage and an abrupt transition from viable to necrotic cells. A representative sample is shown in the image below.
Which of the following is the most likely diagnosis?
CorrectIncorrect -
Question 23 of 38
23. Question
A 76-year-old man comes to the office with a 2-month history of progressive low back pain. The pain is relatively constant and nagging and is unrelieved by rest or position changes. It is especially bad at night and interferes with sleep. The patient has taken acetaminophen and ibuprofen without relief. Medical history is significant for hypertension and osteoarthritis affecting the knees and hands. The patient used intravenous drugs occasionally when he was younger but not for >30 years. Which of the following processes is most likely responsible for this patient’s back pain?
CorrectIncorrect -
Question 24 of 38
24. Question
A 68-year-old man comes to the office due to persistent back pain for the past several months. He is a retired veteran and has not seen a doctor for many years. He has smoked a pack of cigarettes daily for 36 years. His blood pressure is 145/85 mm Hg and pulse is 88/min and regular. Cardiopulmonary examination is unremarkable. The abdomen is soft and nontender. There is focal tenderness over the 10th rib and L1-L2 vertebral region. Neurologic examination is normal. Radionuclide bone imaging reveals increased activity in multiple vertebrae and ribs, corresponding to the patient’s painful areas. Routine radiographs show sclerotic lesions matching the bone scan areas of increased uptake, which are highly suspicious for metastatic cancer. Which of the following additional findings is most likely to be seen in this patient?
CorrectIncorrect -
Question 25 of 38
25. Question
A 68-year-old woman is brought to the emergency department with severe right hip pain after a fall. She tripped on a rug and fell against a wall and then to the floor, landing on her right hip. The patient has a history of depression and polymyalgia rheumatica. Medications include sertraline and low-dose prednisone. She also has a 40-pack-year smoking history. Blood pressure is 145/85 mm Hg and pulse is 96/min. The patient appears to be in significant pain. She is unable to move the right hip, and the right leg appears shorter than the left. A pelvic x-ray is shown in the image below:
Injury involving which of the following arteries is most likely to lead to osteonecrosis in this patient?
CorrectIncorrect -
Question 26 of 38
26. Question
A 2-day-old girl is brought to the office for her initial newborn visit. The patient was born at 39 weeks gestation via a difficult home delivery. Birth weight was 4 kg (8 lb 13 oz). The parents report that the infant has been breastfeeding well but has been only minimally moving her right arm compared to the left. Vital signs are normal. On examination, the right upper extremity is held in adduction and internal rotation with the elbow extended, forearm pronated, and wrist flexed. Moro and biceps reflexes are absent in the right arm; grasp reflex is intact. The remainder of the examination is normal. Which of the following is the most likely site of this patient’s injury?
CorrectIncorrect -
Question 27 of 38
27. Question
A 92-year-old woman is being evaluated due to the presence of skin blisters for the past week. The patient is bed-bound and fatigued. Vital signs are within normal limits. She has no blisters or erosions in the mouth. Skin findings are shown in the image below:
Serologic studies are most likely to show an autoantibody directed against which of the following components?
CorrectIncorrect -
Question 28 of 38
28. Question
A 41-year-old man comes to the office for evaluation of right hip and groin pain. He is an avid bicyclist and has been unable to ride for the past 3 months due to increasing pain exacerbated by cycling or walking. The patient does not recall any recent trauma; ibuprofen has not relieved the pain. Medical history is notable for systemic lupus erythematosus. Temperature is 36.7 C (98 F). On physical examination, the patient has a noticeable limp. There is decreased rotation of the right hip, and both active and passive range of motion are limited due to pain. An x-ray of the hip is inconclusive. MRI of the pelvis is shown in the image below.
Which of the following is the most likely cause of the patient’s pain?
CorrectIncorrect -
Question 29 of 38
29. Question
A 35-year-old man comes to the office due to recurrent foot pain. In the last 6 months, the patient has had 3 episodes of pain at the base of the left great toe. The pain awakens him from sleep and is associated with redness and swelling but no fever. The first 2 episodes resolved spontaneously, but the current episode has been more severe, and ibuprofen has not provided adequate pain relief. The patient has a history of prediabetes and takes metformin. He works as a butcher and spends much of his time in an air-conditioned room. His diet consists primarily of fried foods and salty, carbohydrate-rich snack foods. He drinks 8-10 cups of coffee and 2 L of diet soft drinks daily and 3 or 4 beers nightly. The patient does not use tobacco or illicit drugs. Temperature is 37.5 C (99.5 F), blood pressure is 142/86 mm Hg, and pulse is 92/min. BMI is 29 kg/m2. Examination shows erythema and tenderness at the left first metatarsophalangeal joint. Aspiration of joint fluid reveals many leukocytes and scattered needle-shaped, negatively birefringent crystals. Which of the following is most likely to have contributed to this patient’s condition?
CorrectIncorrect -
Question 30 of 38
30. Question
A 45-year-old man participates in a research study. He undergoes percutaneous skeletal muscle biopsy. Microscopic analysis shows his type I and type II muscle fibers are clustered in groups in contrast to a normal intermixed pattern as shown below.
Which of the following is the most likely cause of the changes observed in this patient’s skeletal muscle?
CorrectIncorrect -
Question 31 of 38
31. Question
A 67-year-old woman comes to the office due to polymyalgia rheumatica. She was diagnosed a year ago and has required supraphysiologic doses of prednisone to control her symptoms. The patient has no other medical conditions. She has adequate intake of calcium and vitamin D from her diet and supplements. The patient does not use tobacco or alcohol. Physical examination is unremarkable. Laboratory evaluation shows normal serum creatinine, 25-hydroxyvitamin D, and calcium levels. Bone density measured by dual x-ray absorptiometry (DXA) reveals a significant decline when compared to her bone density measured at age 65. Which of the following is most likely contributing to this patient’s bone loss?
CorrectIncorrect -
Question 32 of 38
32. Question
A 68-year-old woman comes to the office due to a burning sensation in her chest and throat for the past 2 weeks. Associated symptoms include trouble swallowing. Medical history is significant for osteoporosis, and she has no known drug allergies. The patient has smoked half a pack of cigarettes daily for 50 years, and does not use alcohol or illicit drugs. Temperature is 36.7 C (98 F), blood pressure is 110/70 mm Hg, and pulse is 70/min. BMI is 20 kg/m2. Cardiopulmonary examination shows clear lungs and normal S1 and S2. The abdomen is soft and nontender. Laboratory results are normal. It is determined that the patient’s current symptoms are caused by one of her medications, which is discontinued. Her symptoms subsequently resolve. The medication responsible for this patient’s presentation is also associated with which of the following side effects?
CorrectIncorrect -
Question 33 of 38
33. Question
A group of investigators is studying osteoprotegerin function in patients with postmenopausal osteoporosis. Using an engineered mouse strain, they develop a chimeric monoclonal antibody that blocks binding of receptor activator of nuclear factor kappa B (RANK) to RANK ligand (RANK-L). The investigators inject this monoclonal antibody into human subjects. Eight weeks later, a bone biopsy is obtained from all subjects. Which of the following is the most likely observed finding?
CorrectIncorrect -
Question 34 of 38
34. Question
A 33-year-old woman comes to the office due to swelling affecting her extremities. Over the last 12 months her fingers, arms, and legs have become progressively swollen and somewhat itchy. Physical examination shows diffuse skin thickening extending proximally to the arms, thighs, and trunk. A skin biopsy is performed. Histologic sections show expansion of the dermal layer with atrophy of the intradermal adipose tissue and dermal appendages. High-power images of the dermis demonstrate extensive deposition of dense, eosinophilic connective tissue. This patient is most at risk for which of the following conditions?
CorrectIncorrect -
Question 35 of 38
35. Question
A 34-year-old woman comes to the office due to a 1-year history of pain and swelling in her wrists and hands. The patient has no other medical conditions and takes no medications. Vital signs are normal. No wrinkles are noted on the face and neck. A photograph of her left hand is shown in the exhibit. Interplay among which of the following cells is most likely responsible for this patient’s condition?
CorrectIncorrect -
Question 36 of 38
36. Question
A 17-year-old girl is brought to the office for evaluation of a right wrist mass. She noticed swelling on the top of her wrist approximately 3 months ago but said that it did not bother her until she was teased about it in school. She has no other concerns. Examination shows a nontender, rounded mass on the dorsal wrist that transilluminates with a penlight. An image of the mass is shown below:
Which of the following is the most likely underlying etiology?
CorrectIncorrect -
Question 37 of 38
37. Question
A 24-year-old man comes to the office due to severe pain in the posterior aspect of the right foot that interferes with walking. For the last several months, the patient has also experienced a dull backache and morning stiffness that is relieved by over-the-counter ibuprofen. Family history is insignificant, and he does not use tobacco, alcohol, or illicit drugs. Which of the following is the most probable location of the pathologic process responsible for this patient’s foot pain?
CorrectIncorrect -
Question 38 of 38
38. Question
A 38-year-old man comes to the office due to a 2-month history of an enlarging, localized swelling on the left foot. Tying his shoelaces too tight leads to localized pain and numbness in the third and fourth toes. Examination shows a firm, nontender, well-circumscribed lesion on the dorsum of the foot as shown in the exhibit. The lesion transilluminates when a penlight is applied to it. Without treatment, which of the following is most likely to occur in this patient?
CorrectIncorrect